Anybody wants to shed some light on the LAWGIC aspect of this question?
Apparently, according to PowerScore explanation, the stimulus is a bi-conditional relationship because of the phrase "**if** it satisfies two **requirements**".
...
I chose B because I thought it weakened the argument showing that it does not matter the amount a person digesting the fiber thus weakening Kyra's claim.
Can someone explain A and B to me?
C) being economical and readily available ...
https://7sage.com/lsat_explanations/lsat-54-section-4-question-09/
I've spent about 2 hrs dissecting this question and I just DO NOT understand it. At all. I've looked at discussion boards, books, everything. Please, for the love of God, HELP!
I'm really having trouble with this question. Am I supposed to approach it as a sufficient assumption question though it's a flaw question? Thanks in advance!
Even though this question is old, it has several lessons built into it. I was able to parse this question out mainly because of the lessons on 7Sage. The first lesson I see with this question is the importance of being attuned to the grammar of the LSAT ...
Can someone help me understand the following phrase within the stimulus?
"This willingness is even more revealing than is good-natured acquiescence in having others poke fun at one".
This might be a silly question but for D... It says "A scientific model that contains **many** elements is not a good theory"
And, on the premise, in order to be a good scientific theory your model needs to be simple enough to ...
This is killing me! Why can JNOP be a correct answer just because we happened to pick J first? Okay, if J is the necessary then O and N can go in too and also because they are OR groups and so both can be in S okay. But then why can't be ...
So I was stuck between A and B. Had I not seen this was a 3/5 difficulty I would have chosen A without hesitation. I still choose A, but contemplated B quite a bit.
Can someone please explain how A is wrong and B is ...
I'm thinking that maybe C is wrong because the mergers does no good to the market share, and the newly merged companies still have to gain market share by purchasing competitors. However, D indicates that since some producers have been squeezed out of the ...
I got number 23 right at first, but wrong after a blind review. I don't understand why answer choice D is not a necessary assumption. Isn't it important that the brains of twins aren't any more likely to suffer from ...
I have been reading through some of the previous posts regarding this question, and they have really helped me understand the stimulus and why it is wrong. Basically, the author gave us the following premise:
This question took me a while to understand why A was not the answer. Anson concludes that Dr. Ladlow isn't a responsible psychologist. The question stem asks: "Anson bases his conclusion about Dr. Ladlow based on which of the ...
I understand why the answer was D, but need a good reason to keep me from picking an answer like A. in the future....I was going back and forth between A and D and selected A last minute.
How is D the answer? Why could genetics not be a possible way to determine schizophrenia in the future? - I selected C, but now see why that wasn't right, but not why D is the correct AC... #Help
Generally feel like these types of questions should be easy points, but the translations into lawgic tend to get me crossed-up here. Can someone provide a solid explanation for the ACs? Thanks in advance.